Question
please answer all 6 questions
A monochromatic laser beam of intensity le=553 W/m2 is polarized in the y-direction and propagates in the positive z directio
3) What is the ratio of Ey, final, the maximum value of the y-component of the electric field immediately following the last
image.png
0 0
Add a comment Improve this question Transcribed image text
Answer #1

1) The intensity of the beam immediately following the polarizer: 1 = = 53 = 276.5 W 2) Applying the Malus law: 12 = 1; cos=

Add a comment
Know the answer?
Add Answer to:
please answer all 6 questions A monochromatic laser beam of intensity le=553 W/m2 is polarized in...
Your Answer:

Post as a guest

Your Name:

What's your source?

Earn Coins

Coins can be redeemed for fabulous gifts.

Not the answer you're looking for? Ask your own homework help question. Our experts will answer your question WITHIN MINUTES for Free.
Similar Homework Help Questions
  • Polarizers and a Quarter-Wave Plate

    A monochromatic laser beam of intensity Io = 659 W/m2 is polarized in the y-direction and propagates in the positive z-direction. This beam is incident upon a quarter-wave plate whose fast axis makes an angle of 45 degrees with the x-axis as shown. Following the quarter-wave plate are two polarizers; the transmission axis of the first polarizer is aligned with the x-axis, while the transmission axis of the second polarizer makes an angle of θ1 = 72 degrees with the...

  • A linearly polarized electromagnetic wave has an average intensity of 324 W/m2. This wave is directed...

    A linearly polarized electromagnetic wave has an average intensity of 324 W/m2. This wave is directed towards two ideal polarizers (in real polarizers, transmission is also effected by reflection and absorption). Polarizer A is oriented with its transmission axis at an angle of θ1 = 40.0° with the incident electric field. Polarizer B has its axis at an angle of θ2 = 70.6° with the incident electric field, as shown in the figure. What is the average intensity of the...

  • 8. A polarized light that has an intensity I0 = 62.0 W/m² is incident on three...

    8. A polarized light that has an intensity I0 = 62.0 W/m² is incident on three polarizing disks whose planes are parallel and centered on a common axis. Suppose that the transmission axis of the first polarizer is rotated 14° relative to the axis of polarization of the incident light, and that the transmission axis of each additional analyzer is rotated 14° relative to the transmission axis of the previous one. Calculate the transmitted intensity through all polarizers. _________ W/m2

  • A beam of polarized light has an average intensity of 15 W/m2 and is sent through...

    A beam of polarized light has an average intensity of 15 W/m2 and is sent through a polarizer. The transmission axis makes an angle of 34° with respect to the direction of polarization. Determine the rms value of the electric field of the transmitted beam.

  • law: I = I0 cos²θ where I0 is the intensity of the polarized light beam just before entering the polarizer

    law: I = I0 cos²θ where I0  is the intensity of the polarized light beam just before entering the polarizer, I is the intensity of the transmitted light beam immediately after passing through the polarizer, and is the angular difference between the polarization angle of the incident beam and the transmission axis of the polarizer. After passing through the polarizer, the transmitted light is polarized in the direction of the transmission axis of the polarizing filter. Part DOne way to produce a beam of polarized...

  • A beam of unpolarized sunlight with intensity of 14 W/m2 goes through a ideal polarizer that...

    A beam of unpolarized sunlight with intensity of 14 W/m2 goes through a ideal polarizer that has a transmission axis rotated 30° to the vertical. What is intensity of the beam after it goes through the polarizer? Unpolarized light Polarizer E Transmission axis Polarized light

  • Unpolarized light with an intensity of 10.8 W/m2 is incident on a pair of polarizers. The...

    Unpolarized light with an intensity of 10.8 W/m2 is incident on a pair of polarizers. The transmission axis of the first polarizer makes an angle 39.3° to the vertical, while the transmission axis of the second polarizer is vertical. (b) Suppose the second polarizer is rotated so that its transmission axis is horizontal. What is the intensity of the light after it passes through both polarizers?

  • I only need question #5 answered. I just posted the others in case they are helpful...

    I only need question #5 answered. I just posted the others in case they are helpful in getting the solution to #5. Polarizers An unpolarized beam of intensity lo 413 W/m2 travels in the positive z-direction and is incident from the left on a series of three linear polarizers as shown. The transmission axis of the first polarizer is aligned with the y-axis. The following polarizers make angles of 62 55 degrees, and 63 - 37 degrees with the positive...

  • 4) a) An unpolarized beam of light with intensity 1000 W/m2 is incident on polarizer P, an ideal ...

    4) a) An unpolarized beam of light with intensity 1000 W/m2 is incident on polarizer P, an ideal linear polarizer with transmission axis along the x-axis. The transmitted light is then incident on a second ideal inear polarizer P2, which has transmission axis oriented 50 with respect to Pi. What is the intensity of the light transmitted through P2? b) Now another ideal linear polarizer Ps is inserted between Pi and P2. Its transmission axis is rotated halfway between that...

ADVERTISEMENT
Free Homework Help App
Download From Google Play
Scan Your Homework
to Get Instant Free Answers
Need Online Homework Help?
Ask a Question
Get Answers For Free
Most questions answered within 3 hours.
ADVERTISEMENT
ADVERTISEMENT
ADVERTISEMENT